summaryrefslogtreecommitdiff
path: root/exercise02
diff options
context:
space:
mode:
authorPhilipp Le <philipp-le-prviat@freenet.de>2020-05-08 01:03:00 +0200
committerPhilipp Le <philipp-le-prviat@freenet.de>2021-03-04 01:16:19 +0100
commitb08c6696547280a87871ac29ed0ff5a849637d0d (patch)
treebd7531e0a1c2b8cc6a9f5836172ab77e4b78839b /exercise02
parent0ed9259fa21d69b9c77390d150a6e623e638ae75 (diff)
downloaddcs-lecture-notes-b08c6696547280a87871ac29ed0ff5a849637d0d.zip
dcs-lecture-notes-b08c6696547280a87871ac29ed0ff5a849637d0d.tar.gz
dcs-lecture-notes-b08c6696547280a87871ac29ed0ff5a849637d0d.tar.bz2
WIP: Chapter 2
Diffstat (limited to 'exercise02')
-rw-r--r--exercise02/exercise02.tex83
1 files changed, 82 insertions, 1 deletions
diff --git a/exercise02/exercise02.tex b/exercise02/exercise02.tex
index d8ad77a..dcd06d2 100644
--- a/exercise02/exercise02.tex
+++ b/exercise02/exercise02.tex
@@ -37,4 +37,85 @@
\end{tasks}
\end{solution}
-% Exercise: Is a sine wave with DC bias mono-chromatic -> no \ No newline at end of file
+\begin{question}[subtitle={Using the Fourier transform}]
+ Derive the Fourier transform, without using the duality, of
+ \begin{tasks}
+ \task
+ Derive the Fourier transform of the time shift, without using the duality!
+ \begin{equation*}
+ \mathcal{F}\left\{\underline{f}(t - t_0)\right\}
+ \end{equation*}
+
+ \task
+ Derive the Fourier transform of the frequency shift, without using the duality!
+ \begin{equation*}
+ \mathcal{F}\left\{e^{j \omega_0 t} \underline{f}(t)\right\}
+ \end{equation*}
+
+ %\task
+ %Derive the Fourier transform of the frequency shift using the time shift and duality!
+ \end{tasks}
+\end{question}
+
+\begin{solution}
+ \begin{tasks}
+ \task
+ Let
+ \begin{equation*}
+ \underline{h}(t) = \underline{f}(t - t_0)
+ \end{equation*}
+ The Fourier transform:
+ \begin{equation*}
+ \mathcal{F}\left\{\underline{h}(t)\right\} = \int\limits_{t = -\infty}^{\infty} \underline{f}(t - t_0) \cdot e^{-j \omega t} \, \mathrm{d} t
+ \end{equation*}
+ Substitute $t' = (t - t_0)$ in the integral.
+ \begin{equation*}
+ \mathcal{F}\left\{\underline{h}(t)\right\} = \int\limits_{t' = -\infty}^{\infty} \underline{f}(t') \cdot e^{-j \omega (t' + t_0)} \, \mathrm{d} t'
+ \end{equation*}
+ $e^{-j \omega t_0}$ is a constant.
+ \begin{equation*}
+ \mathcal{F}\left\{\underline{h}(t)\right\} = e^{-j \omega t_0} \underbrace{\int\limits_{t' = -\infty}^{\infty} \underline{f}(t') \cdot e^{-j \omega t'} \, \mathrm{d} t'}_{= \mathcal{F}\left\{\underline{f}(t)\right\} }
+ \end{equation*}
+
+ \task
+ Let
+ \begin{equation*}
+ \underline{h}(t) = e^{j \omega_0 t} \underline{f}(t)
+ \end{equation*}
+ The Fourier transform:
+ \begin{equation*}
+ \mathcal{F}\left\{\underline{h}(t)\right\} = \int\limits_{t = -\infty}^{\infty} e^{j \omega_0 t} \underline{f}(t) \cdot e^{-j \omega t} \, \mathrm{d} t
+ \end{equation*}
+ Factor out $j t$ in the $e$-function.
+ \begin{equation*}
+ \mathcal{F}\left\{\underline{h}(t)\right\} = \int\limits_{t = -\infty}^{\infty} \underline{f}(t) \cdot e^{-j (\omega - \omega_0) t} \, \mathrm{d} t
+ \end{equation*}
+ Substitute $\omega' = \omega - \omega_0$ in the integral.
+ \begin{equation*}
+ \mathcal{F}\left\{\underline{h}(t)\right\} = \underbrace{\int\limits_{t = -\infty}^{\infty} \underline{f}(t) \cdot e^{-j \omega' t} \, \mathrm{d} t}_{= \mathcal{F}\left\{\underline{f}(t)\right\}}
+ \end{equation*}
+ \begin{equation*}
+ \mathcal{F}\left\{\underline{h}(t)\right\} = \underline{F}\left(j \omega' \right) = \underline{F}\left(j \left(\omega - \omega_0\right) \right)
+ \end{equation*}
+
+% \task
+% Let
+% \begin{equation*}
+% \underline{g}(t) = \underline{f}(t - t_0)
+% \end{equation*}
+% We know from a) that
+% \begin{equation*}
+% \underline{G}\left(\omega \right) = \mathcal{F}\left\{\underline{g}(t)\right\} = e^{-j \omega t_0} \cdot \underline{F}\left(\omega \right)
+% \end{equation*}
+% Now, swap $\omega$ and $t$, swap $t_0$ and $\frac{\omega_0}{2 \pi}$, and assume both $\underline{G}$ and $\underline{F}$ are time-domain functions from now on. $\underline{F}$ now represents the original time-domain function which is shifted in frequency.
+% \begin{equation*}
+% \underline{G}\left(t\right) = e^{- j \frac{\omega_0}{2 \pi} t} \cdot \underline{F}\left(t \right)
+% \end{equation*}
+% We already know $\underline{g}$. Assume that both $\underline{g}$ and $\underline{f}$ are frequency-domain functions now. Therefore, swap $\omega$ and $t$, ans swap $t_0$ and $\frac{2 \pi}{\omega_0}$, too.
+% \begin{equation*}
+% \mathcal{F}\left\{\underline{G}(t)\right\} = 2 \pi \cdot \underline{g}\left(- \omega\right) = \underline{f}\left(- \omega + \omega_0\right) = \underline{f}\left(\omega - \omega_0\right)
+% \end{equation*}
+%
+% We obtain the same result as in b). The duality works. \acs{QED}
+ \end{tasks}
+\end{solution}